0

Proof:

Since we have $n!\geq \left(n/2\right)^{n/2} \to \infty$, we can conclude that $\lim\limits_{n\to\infty}\sqrt[n]{n!}=\infty$. I understand that by taking the n-th root we have $\sqrt{n!}\geq n/2$. But why is the inequality true? And is it enough to be "equal or greater than" if solely for $n=0$ the expression $0!=(0/2)^{0/2}$ match?

Analysis
  • 2,450
  • Stirling gives an easy proof. – zhw. Jun 18 '19 at 22:28
  • 2
    I am confused: the title of your post is about proof of $\lim \frac{n}{\sqrt[n]{n!}}$ while the body asks something about $n!\ge (n/2)^{n/2}$. Which proof are you exactly referring to? Could you give the whole proof that you are concerning? The proof-explanation means you want explanation for some established proof. Are you actually asking how to prove the limit in the title? If yes, then see this: https://math.stackexchange.com/q/201906/9464 –  Jun 18 '19 at 22:28
  • I am reffering to a post here by sdcvvc https://math.stackexchange.com/questions/136626/lim-limits-n-to-infty-sqrtnn-is-infinite – Analysis Jun 18 '19 at 22:50
  • The post by sdcvvc doesn't talk about the $\lim = e$ limit, so your title is inaccurate. Your OP also did not link to that post. Anyway, what part of sdcvvc's explanation was hard to follow? As they said, half of the factors in $n\cdots(3)\cdot(2)(1)$ are greater than $n/2$ (rounded down, technically), so the product is greater than $n/2$ rounded down to the power of (how many is half of the factors of $n$). And no, it doesn't matter what happens at $n=0$ if you're talking about the limit $n\to\infty$. – anon Jun 19 '19 at 00:46

2 Answers2

1

I will show by completely elementary means that

$1+1/m \lt \dfrac{(m!)^{1/m}}{m/e} \lt (1+3/\sqrt{m})(1+1/m^2)(1+1/m) $.

I will first show that if $a_n = (1+1/n)^n$ and $b_n = (1+1/n)^{n+1}$ then $a_n < a_{n+1}$ and $b_n > b_{n+1}$.

Since $a_n < b_n$ and $b_n-a_n =\frac1{n}(1+1/n)^n \to 0 $, this shows that $\lim_{n \to \infty} a_n$ and $\lim_{n \to \infty} b_n$ exist and are equal.

We use the very ingenious proof in N.S Mendelsohn, "An application of a famous inequality", Amer. Math. Monthly 58 (1951), 563, which uses the AGMI, which we will use in the form $((v_1+v_2+...v_n)/n)^n > v_1v_2...v_n$ (all $v_i$ positive) with equality if and only if all the $v_i$ are equal (this allows us to avoid the use of n-th roots).

For $a_n$, consider $n$ values of $1+1/n$ and 1 value of 1. By the AGMI, $((n+2)/(n+1))^{n+1} > (1+1/n)^n $, or $(1+1/(n+1))^{n+1} > (1+1/n)^n $, or $a_{n+1} > a_n $.

For $b_n$, consider $n$ values of $1-1/n$ and 1 value of 1. By the AGMI, $(n/(n+1))^{n+1} > (1-1/n)^n$ or $(1+1/n)^{n+1} < (1+1/(n-1))^n$ , or $b_n < b_{n+1}$.

Since the $a_n$ are increasing, the $b_n$ are decreasing, and $a_n < b_n$, all the $a_n$ are less than any of the $b_n$. Since $b_5 = 2.9859... < 3 $, all the $a_n$ are less than 3.

If n > m, since $a_n < b_m$ and $b_m-a_m = a_m/m $, $a_n-a_m < b_m-a_m = a_m/m < 3/m $.

The common limit is called $e$.

Since $a_n < e < b_n$ for all $n$,

$\begin{array}\\ e^m &\gt \prod_{n=1}^m a_n\\ &= \prod_{n=1}^m (1+1/n)^n\\ &= \prod_{n=1}^m \dfrac{(n+1)^n}{n^n}\\ &= \dfrac{\prod_{n=1}^m (n+1)^n}{\prod_{n=1}^m n^n}\\ &= \dfrac{\prod_{n=2}^{m+1} n^{n-1}}{\prod_{n=1}^m n^n}\\ &= \dfrac{\prod_{n=2}^{m+1} n^{n-1}}{m!\prod_{n=1}^m n^{n-1}}\\ &= \dfrac{(m+1)^m}{m!}\\ \text{so}\\ m! &\gt \dfrac{(m+1)^m}{e^m}\\ \text{or}\\ (m!)^{1/m} &\gt \dfrac{m+1}{e}\\ \text{or}\\ \dfrac{(m!)^{1/m}}{m/e} &\gt 1+1/m\\ \end{array} $

Similarly,

$\begin{array}\\ e^m &\lt \prod_{n=1}^m b_n\\ &= \prod_{n=1}^m (1+1/n)^{n+1}\\ &= \prod_{n=1}^m \dfrac{(n+1)^{n+1}}{n^{n+1}}\\ &= \dfrac{\prod_{n=1}^m (n+1)^{n+1}}{\prod_{n=1}^m n^{n+1}}\\ &= \dfrac{\prod_{n=2}^{m+1} n^{n}}{\prod_{n=1}^m n^{n+1}}\\ &= \dfrac{\prod_{n=2}^{m+1} n^{n}}{m!\prod_{n=1}^m n^{n}}\\ &= \dfrac{(m+1)^{m+1}}{m!}\\ \text{so}\\ m! &\lt \dfrac{(m+1)^{m+1}}{e^m}\\ &= \dfrac{(m+1)(m+1)^{m}}{e^m}\\ \text{or}\\ (m!)^{1/m} &\lt \dfrac{(m+1)^{1/m}(m+1)}{e}\\ \text{or}\\ \dfrac{(m!)^{1/m}}{m/e} &\lt (m+1)^{1/m}(1+1/m)\\ &= m^{1/m}(1+1/m)^{1/m}(1+1/m)\\ \end{array} $

If $(1+1/m)^{1/m} = 1+h_m$, $1+1/m =(1+h_m)^m \gt 1+mh_m $ or $h_m < 1/m^2$ so $(1+1/m)^{1/m} \lt 1+1/m^2 $.

Since $(1+m^{-1/2})^m \gt 1+m^{1/2} \gt m^{1/2} $, raising to the $2/m$ power, $m^{1/m} \lt (1+m^{-1/2})^2 =1+2m^{-1/2}+m^{-1} \le 1+3m^{-1/2} $.

Therefore $\dfrac{(m!)^{1/m}}{m/e} \lt (1+3m^{-1/2})(1+1/m^2)(1+1/m) $.

marty cohen
  • 107,799
0

note that from Sterling's approximation: $$n!\sim\sqrt{2\pi n}\left(\frac ne\right)^n$$ and so: $$n(n!)^{-1/n}\sim n(2\pi n)^{-1/2n}\frac{e}{n}=e(2\pi n)^{-1/2n}$$ and we can now say: $$\lim_{n\to\infty}e(2\pi n)^{-1/2n}=\lim_{n\to\infty}\frac{e}{(2\pi)^{1/n}}=e$$

Hyperplane
  • 11,659
Henry Lee
  • 12,215
  • The actual question is not clear. Is OP asking about why $n!\ge (n/2)^{n/2}$. There is nothing showing the rest of the proof other than the outcome in the title and no source is cited – Henry Lee Jun 18 '19 at 23:01